What is the correct expansion of (2x+3) (2x^2-5)

Answers

Answer 1

Answer:

4x^3 + 6x^2 -10x - 15

Step-by-step explanation:

* means multiply

4x^3 + 6x^2 -10x - 15

2x * 2x^2 = 4x^3

2x * -5 = 10x

3 * 2x^2 = 6x^2

3 * -5 = -15

Answer 2

Answer:

4x³ + 6x² - 10x - 15

Step-by-step explanation:

(2x+3) (2x²-5)

4x³ - 10x + 6x² - 15

4x³ + 6x² - 10x - 15


Related Questions

Select the expression that represents the following statement: add 24 to the quotient of 16 and 8.

Answers

Answer:

16/8 + 24

Step-by-step explanation:

What’s the equation of the line that passes through the point (-4,4) and has a slope of 3/4

Answers

Answer:

y-y1=m(x-x1)

y-4=3/4(x+4)

y=3/4x+7

The formula for the circumference of a circle is R = c/2(pi)

Find the radius of a circle that has a circumference of 16(pi)

A) r = 4
B) r = 8
C) r = 12
D) r = 16

Answers

The answer is B) r= 8

The product of integers a,b,c and d is 546 and if 1<a<b<c<d, what is the value of b+c?​

Answers

Hello,

546=2*3*7*13

a=2, b=3,c=7,d=13

b+c=3+7=10

Explain how the quotient of powers was used to simplify this expression

Answers

Answer:

See explanation

Step-by-step explanation:

Given:

5⁴/25 = 5²

25 = 5²

Then,

5⁴ / 25

= 5⁴/5²

Note:

• multiplication sign means addition in indices

• Division sign means subtraction in indices

Both numerator and denominator have the same base, so you'll pick one of the bases and subtract the powers

So,

5⁴/5²

= 5^(4 - 2)

= 5^(2)

= 5²

Therefore,

5⁴ / 25 = 5²

find the slope of the line passing through the points (-2,5) and (3/2,2)

Answers

Answer:

slope = - [tex]\frac{6}{7}[/tex]

Step-by-step explanation:

Calculate the slope m using the slope formula

m = [tex]\frac{y_{2}-y_{1} }{x_{2}-x_{1} }[/tex]

with (x₁, y₁ ) = (- 2, 5) and (x₂, y₂ ) = ([tex]\frac{3}{2}[/tex], 2)

m = [tex]\frac{2-5}{\frac{3}{2}-(-2) }[/tex]

   = [tex]\frac{-3}{\frac{3}{2}+2 }[/tex]

   = [tex]\frac{-3}{\frac{7}{2} }[/tex]

  = - 3  × [tex]\frac{2}{7}[/tex]

  = - [tex]\frac{6}{7}[/tex]

pls help me don't know what to do

Answers

Answer:

x=15

Step-by-step explanation:

The 60 degree angle and the (x+45) degree angle are both the same degree because they are vertical angles.

So to solve, just subtract 45 from 60

60-45=15

That's your answer!

Hope this helps!

What type of health screening would this patient most likely receive?
Sue is a 45-year-old woman with a family history of breast cancer. Her healthcare professional will most likely recommend that she receive a .

Answers

Answer:

A mammogram is what she would receive

Step-by-step explanation:

A cinema is doing a promotion to celebrate their 50th anniversary for 1 week. They give

away a free drink to every 98th customer, a free bag of popcorn to every 112th customer and

a free cinema ticket to every 224th customer. Which lucky customer will be the first to

receive all 3 items?​

Answers

Answer:

1,568 customer

Step-by-step explanation:

Find the lowest common multiple of 98, 112, and 224

98 = 98, 196, 294, 392, 490, 588, 686, 784, 882, 980, 1078, 1176, 1274, 1372, 1470, 1568, 1666

112 = 112, 224, 336, 448, 560, 672, 784, 896, 1008, 1120, 1232, 1344, 1456, 1568, 1680, 1792, 1904

224 = 224, 448, 672, 896, 1120, 1344, 1568, 1792, 2016, 2240

The lowest common multiple of 98, 112, and 224 is 1568

Therefore, the 1,568th customer will be the first to receive all 3 iitem

Find the height h of the triangle.

Answers

Answer:

A, 12 times the square root of 3

Step-by-step explanation:

24 sqaured plus 12 squared=432

square root of 432=20.9

12 times the sqaure root of 3=20.9

What is the value of x?

Answers

Answer:

D. 30°

Step-by-step explanation:

By exterior angle theorem:

x° + 70° = 100°

x° = 100° - 70°

x° = 30°

x = 30

According to the number line, what is the distance between points A and B?

0 6 units
7 units
O 12 units
O 14 units

Answers

Answer:

14 units

Step-by-step explanation:

A = - 2, B = 12

Therefore,

d(A, B) = 12 - (-2) = 12 + 2 = 14 units

PLS HELP SOON WILL MARK BRAINLYEST

A railroad tunnel is shaped like a semi-ellipse, as shown below. A semiellipse is shown on the coordinate plane with vertices on the x axis and one point of intersection with the positive y axis. The height of the tunnel at the center is 35 ft, and the vertical clearance must be 21 ft at a point 8 ft from the center. Find an equation for the ellipse.

Answers

According to the question

b= 35 and (8,21) lies on the ellipse

After calculation we get a= 10

equation for the ellipse.

[tex] \frac{ {x}^{2} }{100} + \frac{ {y}^{2} }{1225} = 1[/tex]

The cost for 2 adults and 5 children to eat at the local buffet restaurant is \$46.50 . The cost for 1 adult and 1 child is $15. What is the cost of a child's meal ?

Answers

Answer:

c = 5.5

Step-by-step explanation:

2 a + 5 c = 46.50

a + c = 15

a = 15 - c

2(15-c ) +5c = 46.50

30 - 2c +5c = 46.5

3c = 16.5

c = 5.5

Let a be the adults and c be the children.

For the first equation there are 2 adults and 5 children so the equation is:-

2a + 5c = 46.50

And for the second equation there is 1 adult and 1 child. So the equation is:-

a + c = 15

a = 15 - c

You take this formula and substitute it to the above formula (2a + 5c = 46.50)

➡️ 2a + 5c = 46.50

➡️ 2(15 - c) + 5c = 46.50

➡️ 30 -2c + 5c = 46.50

➡️ 5c - 2c = 46.50 - 30
(You collect like terms)

➡️ 3c = 16.5

➡️ c = 16.5/3

➡️ c = 5.5

2 1/4 x 3 1/5 brainliest

Answers

Answer:

36/5

Step-by-step explanation:

9/4×16/5

144/20

36/5

hope this is helpful

Answer:

[tex]7\frac{1}{5}[/tex]

Step-by-step explanation:

1. start by turning the fractions improper fractions:

[tex]2\frac{1}{4} =\frac{9}{4}[/tex]

[tex]3\frac{1}{5} =\frac{16}{5}[/tex]

2. then multiply them together:

[tex]\frac{9}{4}[/tex] x [tex]\frac{16}{5}[/tex] = [tex]\frac{144}{20}[/tex]

3. then simplify the fraction:

[tex]\frac{144}{20}[/tex][tex]=\frac{36}{5}[/tex]

4. turn it into a proper fraction:

[tex]\frac{36}{5} =7\frac{1}{5}[/tex]

Hello, Brainly community!

This question is for all of those Calculus people out there.

The volume of a swimming pool is changing with respect to time, such that the volume is given by W(t), where W(t) is measured in cubic centimeters and t is measured in seconds. A tangent line is shown for W(t) at t = 3 seconds. Determine the best estimate for the value of the instantaneous rate of change of W(t) when t = 3.
(I've narrowed down the answer choices to 2, and just really need to find the right way of thinking to find the answer)

(A) W(lim t) as t goes to 3.
(B) [W(3.1) - W(2.9)] / 0.2.

Thank you in advance!

Answers

Answer:

(B)  [tex]\displaystyle \frac{W(3.1) - W(2.9)}{0.2}[/tex]

General Formulas and Concepts:

Calculus

Limits

Derivatives

The definition of a derivative is the slope of the tangent line.

Derivative Notation

Instantaneous Rates

Tangent Line: [tex]\displaystyle f'(x) = \frac{f(b) - f(a)}{b - a}[/tex]

Step-by-step explanation:

Since we are trying to find a rate at which W(t) changes, we must find the derivative at t = 3.

We are given 2 close answer choices that would have the same numerical answer but different meanings:

(A)  [tex]\displaystyle \lim_{t \to 3} W(t)[/tex](B)  [tex]\displaystyle \frac{W(3.1) - W(2.9)}{0.2}[/tex]

If we look at answer choice (A), we see that our units would simply just be volume. It would not have the units of a rate of change. Yes, it may be the closest numerically correct answer, but it does not tell us the rate at which the volume would be changing and it is not a derivative.

If we look at answer choice (B), we see that our units would be cm³/s, and that is most certainly a rate of change. Answer choice (B) is also a derivative at t = 3, and a derivative tells us what rate something is changing.

∴ Answer choice (B) will give us the best estimate for the value of the instantaneous rate of change of W(t) when t = 3.

Topic: AP Calculus AB/BC (Calculus I/I + II)

Unit: Differentiation

Book: College Calculus 10e

(x+4y)^(6)
Can Someone please me how to do this PLEASE!

Answers

The answer is x+4y=-6

Heya Kitties!
What is the value of x?

18x−16=−12x−4



x =

Answers

Answer:

2/5

Step-by-step explanation:

18x+12x=-4+16

30x=12

x=2/5

Answer:

18x-16=-12x-4

18x+12x=-4+16

30x=12

x=12/30

x=2/5

On the unit circle, which of the following angles has the terminal point
coordinates.
A. 45
B. 135
C. 225
D. 315

Answers

Answer: C. 225

Step-by-step explanation:

Decrease £19064.67 by 9.5%

Give your answer rounded to 2 DP. (decimal place)

Please Help !!!

Answers

Answer:

17253.52

Step-by-step explanation:

100-9.5=90.5

19064.67*0.905=17253.52

solve for x. Round to the nearest tenth, if necessary.​

Answers

Answer:

7.1

Step-by-step explanation:

We used SOHCAHTOA because it's a right angle triangle

So because we have an angle with an adjacent of 6.3 and hypotenuse of x

We will use

Cos=adjacent /hypotenuse

7.1 like the last answer if not you can round that


Which expressions are equivalent to 6 - 4.5+ (-8) ?
A 6 + 4.5 + 8
B 6 + 4.5 - 8
C 6 – 4.5'+8
D 6 - 4.5 - 8
E 6+(-4.5) - 8
F 6+(-4.5) + (-8).
G 6+ (-4.5) - (-8)

Answers

Answer:

I'm pretty sure the answers are options D , E and F .

Step-by-step explanation:

So here's how I did it , I got the original expression , solved it and the answer I found is -6.5. So then I solved all the others and D , E and F gave me -6.5.

Hope this helps lol

Which expressions are equivalent c and e are equivalent to 6-4.5+(-8)

jordan wants to save to buy a car and decides to open a banking account that is offering an interest rate of 4.5% compounded annually how much will jordan have in the account after 5 years it he deposits $7,000 today?

Answers

Answer:

8,723.27$

Step-by-step explanation:

1 Find x and H in the right angled below​

Answers

You didn't include the right triangle but this is all I could find.

x = 1

H = sqrt(15)

Greg buys 60 garden plants at a cost price of $2.00 each to sell in his shop. He sells 25 of them at the profit of 75% and 18 of them at the profit of 35%. He sells the rest of the plants for 4/5 of the cost price calculate the profit or loss he makes from selling 60 plants stating if it is a profit or loss

Answers

Answer:

$43.30 profit

Step-by-step explanation:

Total cost of plant:

60*2 = 120

Greg makes total of:

25*(2 + 0.75*2) + 18*(2 + 0.35*2) + (60 - 25 - 18)*2*4/5 = 163.3

Since Greg mare than cost, he has a profit and the amount is:

163.3 - 120 = 43.3

Nathan is 1.55 meters tall. At 1 p.m., he measures the length of a tree's shadow to be 38.15 meters. He stands 32.9 meters away from the tree, so that the tip of his shadow meets the tip of the tree's shadow. Find the height of the tree to the nearest hundredth of a meter .

Answers

Answer:

11.26 m

Step-by-step explanation:

The height of the tree is about 11.25 meters.

What are similar triangles?

When the respective sides are proportional and the corresponding angles are congruent, two triangles are said to be similar.

Given that, the height of the person is 1.55 meters, the length of the tree's shadow is 38.15 meters, and the distance between the person and the tree is 32.9 meters.

Let the height of the tree be x.

Note that the scenario makes two similar triangles.

Since the ratio of the side lengths of similar triangles is proportional, it follows:

(38.15 - 32.9)/1.55 = 38.15/x

5.25/1.55 = 38.15/x

3.39 = 38.15/x

x = 38.15/3.39

x = 11.25

Hence, the height of the tree is about 11.25 meters.

Learn more about similar triangles: https://brainly.com/question/25882965

#SPJ2

What are the solutions to the quadratic equation 3(x – 42 = 752

Answers

X=292.6

Step 1: distribute 3 through parentheses
3x-126=752

Step 2: move the constant to the right and change it sign
3x= 752+126

Step 3: add the numbers
3x= 878

Step 4: divide both sides by three
X= 878/3 or 292.6

5) On each birthday Rosa gets as many roses as she is old in years. She still has all the dried flowers and there are now 120 of them. How old is she? A) 10 B) 12 C) 14 D) 15 E) 20

Answers

Answer:

D) 15

Step-by-step explanation:

This is an arithmatic progression.

The formula for the sum of arithmatic progression is

[tex]s = \frac{n}{2} (2a + (n - 1)d)[/tex]

where d is the common difference between successive terms and a is the first term. By applying this formula,

[tex]120 = \frac{n}{2} (2(1) + (n - 1)(1)) \\ 120 = \frac{n}{2} (1 + n) \\ n(1 + n) = 240 \\ n {}^{2} + n - 240 = 0 \\ (n - 15)(n + 16) = 0 \\ n = 15 \: or \: n = - 16(reject)[/tex]

help and explain//////////////////////////////////////////////

Answers

Answer:

(f + g)(x) = 5x - 3

Step-by-step explanation:

(f + g)(x)

= f(x) + g(x)

= 2x + 4 + 3x - 7 ← collect like terms

= 5x - 3

Answer:

5x - 3

Step-by-step explanation:

(f + g)(x) = f(x) + g(x)

= 2x + 4 + ( 3x - 7 )

= 5x - 3

find the domain of f(x)=sec(2x)

Answers

Answer:

*Refer the image attached

Step-by-step explanation:

*Refer the image attached

Other Questions
does anyone know how to solve? I also have more I need help with. Suppose the economy is in long-run equilibrium. Concerns about pollution cause the government to significantly restrict the production of electricity. At the same time, the value of the dollar falls. What would we expect to happen in the short run? a. The price level will rise, and real GDP might rise, fall, or stay the same. b. The price level will fall, and real GDP might rise, fall, or stay the same. c. Real GDP will rise, and the price level might rise, fall, or stay the same. d. Real GDP will fall, and the price level might rise, fall, or stay the same. Melvin begins his retirement fund at age 30, depositing $1,000 per month until age 50. Cindy begins her retirement fund at age 20, depositing the same $1,000 per month amount until age 50. Both Melvin and Cindy earn 5 percent annual interest on their funds, and there are no tax considerations in this problem. Based on the provided information :____________a) and assuming they retire at age 50, Cindy will have less than 50 percent more than Melvin b) and assuming they retire at age 50. Cindy will have exactly 50 percent more than Melvin. c) and assuming they both retire at age 60. Cindy will have less than Melvin. d) the difference between the two will get larger with higher inflation e) and assuming they retire at age 50. Cindy will have over 50 percent more than Melvin. Two years ago, Global Airlines sold a $250 million bond issue to finance the purchase of new jet airliners. These bonds were issued in at par value with an original maturity of 12 years and a coupon rate of 12%. Determine the value today of one of these bonds to an investor who requires a 14% rate of return on these securities. Is it a discount or premium bond and why Which of the following sentences uses commas correctly? Carol was the last person out of the house wasn't, she? Carol was the last person, out of the house wasn't she? Carol was the last person out of the house, wasn't she? Carol, was the last person out of the house wasn't she? what is the equation of the directx for the following parabola -8(x-5)=(y+1)^2 What should students be aware of all the following red flags when seeking scholarships except Give an example of a substance with an amorphous structure. The standard metric unit of volume is the _____.cubic centimeterlitermillilitercubic meter Joanne's monthly salary is $2400.she spend's of it on food and of it on leisure activities.How much does she have left? please help!!!Simplify3+ 7 X 4 -3/11 + 5/9 can you pls explain this step by step On Monday, Ray had $153.75 in his bank account. On Tuesday, he withdrew $71.00 from his account. After depositing #292.50 on Wednesday, how much money did Ray have in his account As x increases by 1 unit, what is the exponential growth factor?? Given: x + 2 < -5.Choose the solution set.{x | x R, x < -3}{x | x R, x < 3}{x | x R, x < -7}{x | x R, x < 7} Matching Items Reported to Cash Flow Statement Categories (Indirect Method).Adidas AG is a global company that designs and markets sports and fitness products, including footwear, apparel, and accessories. Some of the items included in its recent annual consolidated statement of cash flows presented using the indirect method are listed here. Indicate whether each item is disclosed in the Operating Activities, Investing Activities, or Financing Activities section of the statement or use Not Applicable if the item does not appear on the statement. (Note: This is the exact wording used on the actual statement).1. Dividends paid. 2. Repayments of short-term borrowings. 3. Depreciation and amortization. 4. Proceeds from reissuance of treasury shares to employees. 5. [Change in] Accounts payable and other liabilities. 6. Cash collections from customers. 7. Purchase of investments. 8. Net income. 9. Purchase of property, plant, and equipment. 10. Increase in receivables and other assets. La familia pas el da en el parque. No____a. dio un paseob. jug al bolicheC. compr heladod. rem por el lago He said to me"don't follow me"( change in indirect speech) Express sin U as a fraction in simplest terms Two cities,a and are mapped on the coordinate plane. How far apart are they from each other?